LSAT and Law School Admissions Forum

Get expert LSAT preparation and law school admissions advice from PowerScore Test Preparation.

 est15
  • Posts: 94
  • Joined: Aug 28, 2013
|
#16211
Hi, I have a question about why C is incorrect. I get that if the answer were "safety is often sacrificed in order to reach a goal as quickly, cheaply, and terribly as possible," the answer would definitely be wrong because of the word terribly. But if it simply fails to mention cheaply why is it still wrong? Because isn't quickly and cheaply included within the broader category of quickly? I would think that it wouldn't necessarily be wrong to err on the less specific side for an answer choice, unless it's that principle types of questions have to have an answer choice that's super specific? Thanks.
 Lucas Moreau
PowerScore Staff
  • PowerScore Staff
  • Posts: 216
  • Joined: Dec 13, 2012
|
#16222
Hello, est15,

I believe you are overreaching a little bit. "Cheaply" is not necessarily included within "quickly"...have you ever paid extra to have something done faster? ;) C is incorrect specifically because it says quickly rather than cheaply, as the two problems mentioned are cost and safety, not speed.

Hope that helps,
Lucas Moreau
 srcline@noctrl.edu
  • Posts: 243
  • Joined: Oct 16, 2015
|
#21755
Hello,

I am having a hard time seeing why A is the right answer. I choose C. The question stem is "the passage conforms most closely to which one of the following propositions?

Answer choice C. says that Safety is often sacrificed in order to reach a goal as quickly as possible.

I choose this answer because of the whole second half of the premise, especially when it says "manufacturing flaws that critics contend are a direct consequence of the pressure to produce results as quickly and cheaply as possible.

Thankyou
Sarah
 Adam Tyson
PowerScore Staff
  • PowerScore Staff
  • Posts: 5153
  • Joined: Apr 14, 2011
|
#21759
Good question, Sarah, and I see why C looks attractive to you. For me, the problem with that answer is that it focuses on "quickly" to the exclusion of "cheaply", and the bulk of the stimulus was not about speed but about cost.

A covers the argument more completely, and whether the issue is speed or cost makes no difference. Efforts to solve one problem (keep costs down or go faster) can make another problem (safety) worse. In a sense, if C is true, it forces A to also be true, which means you must eliminate C - we call that the Uniqueness Principle of answers. Anytime one answer forces another to also be correct, the one doing the forcing must be incorrect.

Look at it again in that light, seeing what answer C does to answer A, and see if that helps clear it up.

Keep pounding (says this very happy Panthers fan)!
 srcline@noctrl.edu
  • Posts: 243
  • Joined: Oct 16, 2015
|
#21767
Hello Adam,

Before I posted this question I was thinking the same thing, in that I was also thinking that C allows for A to be true as well, I just didn't know the name for it! Also I read back over the stimulus, I see now that its about cost more so than it is about safety.
Thankyou
Sarah
 avengingangel
  • Posts: 275
  • Joined: Jun 14, 2016
|
#36425
Lucas Moreau wrote:Hello, est15,

I believe you are overreaching a little bit. "Cheaply" is not necessarily included within "quickly"...have you ever paid extra to have something done faster? ;) C is incorrect specifically because it says quickly rather than cheaply, as the two problems mentioned are cost and safety, not speed.

Hope that helps,
Lucas Moreau

Dang! I didn't even realize that I was doing that same thing — equivocating quickly with cheaply.. I need to read more carefully !! Thanks for your help.
 avengingangel
  • Posts: 275
  • Joined: Jun 14, 2016
|
#44420
Is this stimulus just a statement or an argument?? I am used to Most Closely Conforms question stems to be associated with arguments, but I am not sure if this one has a conclusion ?? Thanks.
 Daniel Stern
PowerScore Staff
  • PowerScore Staff
  • Posts: 81
  • Joined: Feb 07, 2018
|
#44437
Avenging:

Although this is considered a "Strengthen" and "Principle" question, and strengthen questions typically do contain arguments, I can see what you mean: every time I try to pin down one part or the other as the conclusion, it seems unclear to me. So I think this stimulus is, as you suspected, merely a list of premises.

So it is almost a hybrid of must-be-true and strengthen, with the principle overlay. Our answer choice will be our conclusion, but because it is a "principle" type question -- we know this from the use of the term "proposition" in the question stem -- the conclusion will be stated in abstract terms rather than in terms that relate directly to the content of the stimulus.

If it were a straight must-be-true, our credited response, A, might read "Attempts to bring down the cost of the space program can lead to further compromise in the safety of space missions." Instead, because it is principle, we get the generalized version -- "Attempts to solve one problem can lead to the exacerbation of another problem."

Good luck in your studies,
Dan
User avatar
 Neil J
  • Posts: 6
  • Joined: Sep 07, 2021
|
#90313
Hi - I understand why A is correct but cannot adequately explain why B is wrong. Could someone help me out? Thank you!
 Jeremy Press
PowerScore Staff
  • PowerScore Staff
  • Posts: 1000
  • Joined: Jun 12, 2017
|
#90353
Hi Neil,

Sure! The problem with answer choice B is that the safety risks in the stimulus were a result of "budgetary pressures." That's different than those risks being a result of focusing on "scientific progress." In fact, the stimulus never gives any indication that "scientific progress" is part of the problem here. To fix answer choice B, you'd need it to say something like, "Safety risks are sometimes ignored in the name of saving money." Make sense?

I hope this helps!

Get the most out of your LSAT Prep Plus subscription.

Analyze and track your performance with our Testing and Analytics Package.